PT2.S4.Q9 - Clark brand name parts

isdmyungisdmyung Member
edited December 2020 in Logical Reasoning 121 karma

I am unsure why D is correct over E. I am assuming that E makes a flaw through its logic statement?

Comments

  • isdmyungisdmyung Member
    121 karma

    To be clear this was PT2 S4 Q9

  • canihazJDcanihazJD Alum Member Sage
    8318 karma

    E is not required and also seems to contradict the stimulus. The argument is only talking about Clark parts. E talks about any parts made for cars manufactured in country... which seems like it would include the foreign made parts the ad is saying are poorly made.

Sign In or Register to comment.